Question

In: Advanced Math

If a set K that is a subset of the real numbers is closed and bounded,...

If a set K that is a subset of the real numbers is closed and bounded, then it is compact.

Solutions

Expert Solution

Let K be a closed and bounded subset of real numbers. Then K can be put inside a closed bounded interval [a,b] and we know that closed subset of compact sets are compact. So it is sufficient to prove that closed bounded intervals are compact.


Related Solutions

Prove Corollary 4.22: A set of real numbers E is closed and bounded if and only...
Prove Corollary 4.22: A set of real numbers E is closed and bounded if and only if every infinite subset of E has a point of accumulation that belongs to E. Use Theorem 4.21: [Bolzano-Weierstrass Property] A set of real numbers is closed and bounded if and only if every sequence of points chosen from the set has a subsequence that converges to a point that belongs to E. Must use Theorem 4.21 to prove Corollary 4.22 and there should...
for every bounded, closed and convex subset K of Y, there exists a retract from Y...
for every bounded, closed and convex subset K of Y, there exists a retract from Y onto K prove the lemma
Give an example of a set A subset the real numbers for which both A and...
Give an example of a set A subset the real numbers for which both A and the complement of A are unbounded.
1. Prove that any compact subset ot a metric space is closed and bounded. 2. Prove...
1. Prove that any compact subset ot a metric space is closed and bounded. 2. Prove that a closed subset of a compact set is a compact set.
Suppose f maps a closed and bounded set D to the reals is continuous. Then the...
Suppose f maps a closed and bounded set D to the reals is continuous. Then the Uniform Continuity Theorem says f is uniformly continuous on D. To prove this, we will suppose it is not true, and arrive at a contradiction. So, suppose f is not uniformly continuous on D. If f is not uniformly continuous on D, then there exists epsilon greater than zero and sequences (a_n) and (b_n) in D for which |a_n -b_n| < 1/n and |f(a_n)...
Which of the following statements are correct? a) If A is a bounded subset of the...
Which of the following statements are correct? a) If A is a bounded subset of the real line, every infinite subset of A has a limit point. b) If A is a bounded subset of the real line, every open cover of A has a finite subcover. c) If A is an infinite open subset of the real line, there is an infinite open cover with a finite subcover. d) If A is a closed subset of the real line,...
Let A be a set of real numbers. We say that A is an open set...
Let A be a set of real numbers. We say that A is an open set if for every x0 ∈ A there is some δ > 0 (which might depend on x0) such that (x0 − δ, x0 + δ) ⊆ A. Show that a set B of real numbers is closed if and only if B is the complement of some open set A
3. Suppose A and B are non-empty sets of real numbers that are both bounded above....
3. Suppose A and B are non-empty sets of real numbers that are both bounded above. (a) Prove that, if A ⊆ B, then supA ≤ supB. (b) Prove that supA∪B = max{supA,supB}. (c) Prove that, if A∩B 6= ∅, then supA∩B ≤ min{supA,supB}. Give an example to show that equality need not hold.
Let A = {a1,...,an} be a set of real numbers such that ai >= 1 for...
Let A = {a1,...,an} be a set of real numbers such that ai >= 1 for all i, and let I be an open interval of length 1. Use Sperner’s Theorem to prove an upper bound for the number of subsets of A whose elements sum to a number inside the interval I.
Let k ≥ 2. Use that R (the real numbers) is complete to show R^k is...
Let k ≥ 2. Use that R (the real numbers) is complete to show R^k is complete.
ADVERTISEMENT
ADVERTISEMENT
ADVERTISEMENT